7
$\begingroup$

The question

This question that arose in a discussion with Ron Adin is quite simple:

For which pairs $k$ and $n$ does $n$ divide ${{n-2} \choose {k}}$?

Simple observations

  1. It is easy to see that for every $k$ there are finitely many possible values of $n$. One way to see it is to note that if $n$ divides ${{n-2} \choose {k}}$ it also divides $(n-2)(n-3)\dotsb (n-k-1)$ and therefore also (if $n \ge k+2$) it must divide $(k+1)!$.

  2. For $n=2k+2$, $\frac {1}{n} {{n-2} \choose {k}}=\frac {1}{2(k+1)}{{2k} \choose {k}}=\frac {1}{2}C_k$, where $C_k$ is the $k$-th Catalan number. So the question is when the $k$-th Catalan number is even. As $C_k={{2k+1} \choose {k}}-2 {{2k}\choose {k+1}}$, this is if and only if ${{2k+1} \choose {k}}$ is even and by Kummer's theorem this always happens, unless $k$ and $k+1$ have no common $1$ digits in base $2$, namely iff $k+1$ is a power of $2$. Maybe a similar analysis can be done in other cases as well, and perhaps a complete description is possible.

Experimenting

a little, it seems that given $n$ there are few values of $k$ such that $n \mid{{n-2} \choose {k}}$.

Motivation

These are the cases where vertex-regular $n$-vertex $k$-dimensional $\mathbb Q$-acyclic complexes with complete $(k-1)$-dimensional skeletons exist. We know (albeit by an indirect argument and not by an explicit construction) that whenever $d_1,d_2,\dotsc d_n$ are non-negative integers that sum up to ${{n-2} \choose {d}}$ then a $\mathbb Q$-acyclic complex with complete $(k-1)$-dimensional skeletons exists such that the degree of vertex $i$ (namely, the number of $k$-faces containing it) is $d_i + {{n-2} \choose {k-1}}$.

Related MO question: Seeking very regular $\mathbb Q$-acyclic complexes

$\endgroup$
4
  • 1
    $\begingroup$ Not an answer, but an empirical observation: For $n=3k+3$ the exceptions are also rare, and in those cases $k$ has a curious pattern in base 3. $\endgroup$ Jun 4, 2021 at 8:25
  • $\begingroup$ Indeed, for $n = \ell (k + 1)$ we ask when the number of lattice paths from $(0,0)$ to $((\ell-1)k, k)$ strictly below $y=x/(\ell-1)$ is divisible by $\ell$. $\endgroup$ Jun 4, 2021 at 10:16
  • 1
    $\begingroup$ Whenever $\ell$ is a prime, it seems that we have divisibility if and only if the digits of $k$ are not weakly increasing in base $\ell$. $\endgroup$ Jun 4, 2021 at 11:06
  • 1
    $\begingroup$ A similar problem stated by Erdos and Graham was investigated in: M. Ulas, A. Schinzel, A note on Erdos–Straus and Erdos-Graham divisibility problems (with an Appendix by Andrzej Schinzel), Int. J. Number Theory vol. 9 (3) (2013), 583-599. $\endgroup$ Jun 6, 2021 at 7:28

1 Answer 1

2
$\begingroup$

I doubt there exists a simple description for the general solution, however it's possible to give a characterization in some cases.

For example, in the case of square-free odd $n$, Lucas' theorem implies that $(n,k)$ is a solution iff for every prime $p\mid n$, there exists a base-$p$ digit in $k$ that is greater than the corresponding base-$p$ digit of $n-2$. For the last digit, it means that $p\mid k+1$.

From here we can construct a particular series of solutions $(n,k)$ with semi-prime $n=pq$, assuming, say, that $p\mid (k+1)$, i.e. the last base-$p$ digit of $k$ is $p-1$, and $p\equiv 1\pmod{q}$, i.e. the second but last base-$q$ digit of $n-2$ is $0$. Taking primes $q$ and $p=qs+1$ and integer $k=pt-1$ for some integers $s>0$ and $t\in[0,q]$, where to satisfy the condition on the second but last base-$q$ digit of $k=pt-1=qst+t-1$, it's enough to have $q\nmid st$. That is, we need to have $q\nmid s$ and $1\leq t\leq q-1$.

Example. Take $q=23$ and $p=2\cdot 23+1=47$ giving $n=pq=1081$. Then any $k\in\{47t-1\ :\ t=1,2,\dotsc,22\}$ will do the job.

$\endgroup$

Your Answer

By clicking “Post Your Answer”, you agree to our terms of service and acknowledge that you have read and understand our privacy policy and code of conduct.

Not the answer you're looking for? Browse other questions tagged or ask your own question.